3
$\begingroup$

For a Schwartz function $\psi(x)=xe^{-x^2}$ define $\varphi(x):=\psi'(x)$ and consider a family of $L^1$-dilations of $\varphi$ given by: $$ \varphi_t(x)=\frac{1}{t}\varphi(x/t), \qquad t>0. $$ $\textbf{Question:}$ Is there a function $f\in L^\infty(\mathbb{R})$ such that \begin{equation}\label{eq:1} \liminf_{t\rightarrow 0^+} |\varphi_t\ast f(0)|>0, \end{equation} where $\ast$ denotes a convolution on $\mathbb{R}$ ?

If not, does exist $f\in L^\infty(\mathbb{R})$ satisfying a weaker condition \begin{equation} \int_0^1 |\varphi_t\ast f(0)|\, \frac{dt}{t}=\infty ? \end{equation}

I editted the question after Aleksei's comment.

$\endgroup$
4
  • 1
    $\begingroup$ Doesn't $\chi_{[0, \infty)}$ work? $\endgroup$ May 5, 2020 at 20:54
  • $\begingroup$ Aleksei, thank you, you're right. However, I had something different on my mind when asking the question. I wanted to state it in a relatively general form and unfortunately I made it trivial. Let me edit it and please have a look at the updated version :] My apologies! $\endgroup$
    – Tony419
    May 5, 2020 at 21:14
  • $\begingroup$ It seems to me I can construct an example for the weaker question. If you are interested I can post it as an answer. $\endgroup$ May 5, 2020 at 21:54
  • $\begingroup$ @AlekseiKulikov, please do. I'm excited to see it! $\endgroup$
    – Tony419
    May 5, 2020 at 21:59

1 Answer 1

2
$\begingroup$

$\newcommand{\ph}{\varphi}$ $\newcommand{\eps}{\varepsilon}$ Let $a_k$ be a very fast-growing sequence of integers (I think $a_k = 2^{1000k}$ should be enough). Consider the function $f$ defined as $$f(x) = \sum_{k = 1}^\infty \chi_{[a_k, 2a_k]}.$$

I claim that there is a constant $c > 0$ such that for $\frac{1}{200a_n} \le t \le \frac{1}{100a_n}$ we have $\ph_t(x)*f(0) \ge c$. This is clearly enough since integral of $\frac{1}{t}$ over each of these intervals is $\log(2)$. We have

$$\ph_t(x)*f(0) = \sum_{k = 1}^{n-1}\int_{ta_k}^{2ta_k}\ph(-x)dx +\int_{ta_n}^{2ta_n}\ph(-x)dx +\sum_{k = n+1}^\infty\int_{ta_k}^{2ta_k}\ph(-x)dx.$$

For the first sum we can bound it by the integral $\int_0^{2ta_{n-1}}|\ph(-x)|dx$, which is at most $2Cta_{k-1}$, where $C = \max\limits_{x\in \mathbb{R}} \ph(x)$ which we can make smaller than any $\eps > 0$ if $\frac{a_{n-1}}{a_n}$ is small enough.

For the third sum again we can bound it by $\int_{ta_{n+1}}^\infty |\ph(-x)|dx$. Since $\int_\mathbb{R} |\phi(s)|ds$ converges we can make it smaller than any $\eps$ as long as $ta_{n+1}$ is big enough, that is $\frac{a_{n+1}}{a_n}$ is big enough.

Finally for the middle term we have $x$ is always at least $\frac{1}{200}$ and at least $\frac{1}{50}$. For these $x$ we have that $\phi(-x)$ is at least some constant $p > 0$ (can be seen from the direct computation although the whole argument essentially works for any nonzero, continuous $L^1$ function).

In total we have $$\ph_t(x)*f(0) \ge ta_np - 2\eps\ge \frac{p}{200} - 2\eps.$$

Choosing $\eps = \frac{p}{1000}$ finishes the story.

As I said in the beginning, exponential grows of $a_k$ is enough for this argument to work so we can actually replace $\frac{1}{t}$ by any decreasing positive weight $w(t)$ with $\int_0^1 w(t)dt = \infty$.

$\endgroup$
1
  • $\begingroup$ Brilliant! Thank you so much for sharing this beautiful argument, Aleksei! $\endgroup$
    – Tony419
    May 6, 2020 at 0:39

Your Answer

By clicking “Post Your Answer”, you agree to our terms of service and acknowledge you have read our privacy policy.

Not the answer you're looking for? Browse other questions tagged or ask your own question.